categories - Category Theory list
 help / color / mirror / Atom feed
* Re: stupid question?
@ 2000-03-29 20:23 Peter Freyd
  2000-03-30  1:32 ` question? Michael Barr
  0 siblings, 1 reply; 3+ messages in thread
From: Peter Freyd @ 2000-03-29 20:23 UTC (permalink / raw)
  To: categories; +Cc: mm.mawanda

 M.M. Mawanda asks:

>I have been asked the following question: Is it true that any function
>defined in a real number closed interval [a,b] (there is not a hypothesis
>of continuity) is bounded in an open  subinterval (c,d) of [a,b]? My
>spontaneous was NO. Unfortunately I cannot find a counter-example to
>disapproved my answer. Can someone help.     

No it is not true. For example, the function defined by:

  f(x) =  if  x  is irrational then  0  else
          if  x =  p/q  where  p  and  q  are co-prime then  q.



^ permalink raw reply	[flat|nested] 3+ messages in thread

* Re:stupid question?
  2000-03-29 20:23 stupid question? Peter Freyd
@ 2000-03-30  1:32 ` Michael Barr
  0 siblings, 0 replies; 3+ messages in thread
From: Michael Barr @ 2000-03-30  1:32 UTC (permalink / raw)
  To: categories

It is not even true for additive functions.  Take a Hamel base and send
every element of the base to 1.

On Wed, 29 Mar 2000, Peter Freyd wrote:

>  M.M. Mawanda asks:
> 
> >I have been asked the following question: Is it true that any function
> >defined in a real number closed interval [a,b] (there is not a hypothesis
> >of continuity) is bounded in an open  subinterval (c,d) of [a,b]? My
> >spontaneous was NO. Unfortunately I cannot find a counter-example to
> >disapproved my answer. Can someone help.     
> 
> No it is not true. For example, the function defined by:
> 
>   f(x) =  if  x  is irrational then  0  else
>           if  x =  p/q  where  p  and  q  are co-prime then  q.
> 




^ permalink raw reply	[flat|nested] 3+ messages in thread

* Re: Re: stupid question?
@ 2000-03-29 23:13 Max Kanovitch
  0 siblings, 0 replies; 3+ messages in thread
From: Max Kanovitch @ 2000-03-29 23:13 UTC (permalink / raw)
  To: categories

 Dear M.M. Mawanda,
 
 > >I have been asked the following question: Is it true that any function
 > >defined in a real number closed interval [a,b] (there is not a hypothesis
 > >of continuity) is bounded in an open  subinterval (c,d) of [a,b]?

 The real fun is about a function f such that
 f is unbounded in any open interval (c,d), and
 in addition to that:     f(x+y) = f(x)+f(y).
 
 > Date: Wed, 29 Mar 2000 15:23:16 -0500 (EST)
 > From: Peter Freyd <pjf@saul.cis.upenn.edu>
 > Subject: categories: Re: stupid question?
 > 
 >  M.M. Mawanda asks:
 > 
 > >I have been asked the following question: Is it true that any function
 > >defined in a real number closed interval [a,b] (there is not a hypothesis
 > >of continuity) is bounded in an open  subinterval (c,d) of [a,b]? My
 > >spontaneous was NO. Unfortunately I cannot find a counter-example to
 > >disapproved my answer. Can someone help.     
 > 
 > No it is not true. For example, the function defined by:
 > 
 >   f(x) =  if  x  is irrational then  0  else
 >           if  x =  p/q  where  p  and  q  are co-prime then  q.
 > 



^ permalink raw reply	[flat|nested] 3+ messages in thread

end of thread, other threads:[~2000-03-30  1:32 UTC | newest]

Thread overview: 3+ messages (download: mbox.gz / follow: Atom feed)
-- links below jump to the message on this page --
2000-03-29 20:23 stupid question? Peter Freyd
2000-03-30  1:32 ` question? Michael Barr
2000-03-29 23:13 Re: stupid question? Max Kanovitch

This is a public inbox, see mirroring instructions
for how to clone and mirror all data and code used for this inbox;
as well as URLs for NNTP newsgroup(s).